Show that u'(t) = r(t).(r'(t)Xr'''(t))

  • Thread starter Thread starter michonamona
  • Start date Start date
Click For Summary
SUMMARY

The discussion focuses on proving the derivative of the vector-valued function u(t) defined as u(t) = r(t) · (r'(t) × r''(t)). The proof shows that u'(t) = r(t) · (r'(t) × r'''(t)), utilizing vector calculus identities. Key techniques include applying the product rule for derivatives and properties of the cross product, specifically that a · (b × c) = c · (a × b) and the anti-commutative property of the cross product. The discussion highlights the importance of understanding these vector identities to simplify the proof effectively.

PREREQUISITES
  • Vector calculus, specifically differentiation of vector functions
  • Understanding of cross product properties in three-dimensional space
  • Familiarity with the product rule for derivatives
  • Basic knowledge of vector-valued functions and their derivatives
NEXT STEPS
  • Study vector calculus identities, particularly the properties of the cross product
  • Practice differentiation of vector-valued functions using the product rule
  • Explore examples of vector calculus problems involving derivatives and cross products
  • Learn about applications of vector calculus in physics and engineering contexts
USEFUL FOR

Students in advanced mathematics or physics courses, particularly those studying vector calculus, as well as educators seeking to enhance their understanding of vector differentiation techniques.

michonamona
Messages
120
Reaction score
0

Homework Statement


Let u(t) be a vector valued function, where

u(t) = r(t).(r'(t)Xr''(t))

where r(t) is a vector valued function, and (r'(t)Xr''(t)) the cross product of the first and second derivative of r(t). Show that

u'(t) = r(t).(r'(t)Xr'''(t))

where r'''(t) is the 3rd derivative of r(t).

Homework Equations





The Attempt at a Solution



I got this question on an exam and did not know how to solve it. I started out by computing the cross product of (r'(t)Xr''(t)) by using determinants, and then took the dot product of r(t).(r'(t)Xr''(t)), which gave me a really ugly vector. I ran out of time while computing r(t).(r'(t)Xr'''(t)). Is there some other technique I could've applied to prove this?

Thanks,
M
 
Physics news on Phys.org
u(t) is a scalar, not vector-valued. You can see that many terms in the derivative of u cancel or vanish by using the identities

\vec{a}\times \vec{b} = - \vec{b} \times {a}

\vec{a}\cdot (\vec{b}\times \vec{c}) = \vec{c} \cdot( \vec{a}\times{b}).
 
I'm using ' for derivative & without t in parantheses.:--

u=r.(r'Xr"), differentiating both sides, use these two formulas: (a.b)'=a'.b + a.b'
(aXb)'=a'Xb + aXb'

u' = r'.(r'Xr") + r.(r'Xr")'
= 0 + r.(r"Xr" + r'Xr''') = r(r'Xr''') since a.(aXb )= 0 and aXa = 0
proved
 
let r=a\vec{i}+b\vec{j}+c\vec{k}

<br /> \begin{bmatrix}<br /> a&#039;b&#039;c&#039;&#039;\ -a&#039;b&#039;&#039;c&#039;\ +b&#039;c&#039;a&#039;&#039;\ -b&#039;c&#039;&#039;a&#039;\ +c&#039;a&#039;b&#039;&#039;\ -c&#039;a&#039;&#039;b&#039; \\ <br /> ab&#039;&#039;c&#039;&#039;\ -ab&#039;&#039;&#039;c&#039;\ +bc&#039;&#039;a&#039;&#039;\ -bc&#039;&#039;&#039;a&#039;\ +ca&#039;&#039;b&#039;&#039;\ -ca&#039;&#039;&#039;b&#039; \\ <br /> ab&#039;c&#039;&#039;&#039;\ -ab&#039;&#039;c&#039;&#039;\ +bc&#039;a&#039;&#039;&#039;\ -bc&#039;&#039;a&#039;&#039;\ +ca&#039;b&#039;&#039;&#039;\ -ca&#039;&#039;b&#039;&#039; <br /> \end{bmatrix}<br /> \\ <br />
It's not a matrix, you should read it as one line addition.yep, it' spretty ugly if you're under stress and hurry.

Row 1 completely disappear 1with 4, 2-5, 3-6
Then
r2c1 with r3c2
r2c3 with r3c4
r2c5 with r3c6

Remain 6 terms, the other equation.
 
Last edited:
Thanks everyone. I understand it now. I should have studied the properties of cross product more closely.

M
 
Question: A clock's minute hand has length 4 and its hour hand has length 3. What is the distance between the tips at the moment when it is increasing most rapidly?(Putnam Exam Question) Answer: Making assumption that both the hands moves at constant angular velocities, the answer is ## \sqrt{7} .## But don't you think this assumption is somewhat doubtful and wrong?

Similar threads

Replies
1
Views
1K
  • · Replies 1 ·
Replies
1
Views
1K
  • · Replies 3 ·
Replies
3
Views
2K
Replies
1
Views
1K
Replies
4
Views
2K
Replies
10
Views
3K
  • · Replies 2 ·
Replies
2
Views
1K
  • · Replies 1 ·
Replies
1
Views
1K
  • · Replies 2 ·
Replies
2
Views
2K
  • · Replies 2 ·
Replies
2
Views
4K